Question

Slove these 2 math step by step

image.png


Two displacement vectors \(\vec{a}\) and \(\vec{b}\) have the components in meters, \(a_{x}=4.4 \mathrm{~cm}, a_{y}=\) \(1.6 \mathrm{~cm}, b_{x}=2.0 \mathrm{~cm}, b_{y}=5.5 \mathrm{~cm} .\) Find the angle between the vectors.

0 0
Add a comment Improve this question Transcribed image text
Answer #1

Salution: Griven a a= 4:4 êt 1.6 9 5 = aît 5.5 * To find o angle between a and to * * we know a. D= 121161 Cosco) where o is

Add a comment
Know the answer?
Add Answer to:
Slove these 2 math step by step
Your Answer:

Post as a guest

Your Name:

What's your source?

Earn Coins

Coins can be redeemed for fabulous gifts.

Not the answer you're looking for? Ask your own homework help question. Our experts will answer your question WITHIN MINUTES for Free.
Similar Homework Help Questions
  • What are (c) the magnitude and (d) the angle (relative to the pos- itive direction of...

    What are (c) the magnitude and (d) the angle (relative to the pos- itive direction of the x axis) of the overall displacement? 15 SSM ILW WWW The two vec- tors a and b in Fig 3-28 have equal1 magnitudes of 10.0 m and the angles are θ,-30° and 105°. Find the (a) x and (b) y components of their vector sum T. (c) the magnitude of T, and (d) the angle T makes with the positive direction of the...

  • The two vectors $\vec{a}$ and $\vec{b}$ in the figure have equal magnitudes of 35.0 m and...

    The two vectors $\vec{a}$ and $\vec{b}$ in the figure have equal magnitudes of 35.0 m and the angles are θ1 = 26° and θ2 = 102°. Find the x-component of their vector sum $\vec{r}$. Incorrect. Find the y-component of their vector sum $\vec{r}$. Find the magnitude of $\vec{r}$? Find the angle $\vec{r}$ makes with the positive direction of the x -axis.

  • Given the two position vectors r1 = (-2.,0, 8.0, -2.0) and r2 = (4.0, -2.0, 2.0),...

    Given the two position vectors r1 = (-2.,0, 8.0, -2.0) and r2 = (4.0, -2.0, 2.0), in meters, find (a) their resultant and (b) the angle the resultant makes with the x-axis.

  • The figure below shows two vectors lying in the xy plane. Determine the signs of the x and y...

    The figure below shows two vectors lying in the \(x y\) plane. Determine the signs of the \(x\) and \(y\) components of \(\overrightarrow{\mathrm{A}}, \overrightarrow{\mathrm{B}},\) and \(\overrightarrow{\mathrm{A}}+\overrightarrow{\mathrm{B}},\) and place your answers in the following table.

  • Name of All Group members Section At the end of this activity you will be able...

    Name of All Group members Section At the end of this activity you will be able to: a. Add and Subtract vectors b. Resolve vectors into horizontal and vertical components c. Apply vector components to solve kinematic problems. Rank the magnitudes of the vectors from greatest to least. If two or more vectors have the same magnitude, the sequence in which you list them does not matter. 7Least Greatest 1 2 3 , 4 2. List all of the vectors...

  • Vector Manipulation For the vectors shown below, determine the values of A and B that would...

    Vector Manipulation For the vectors shown below, determine the values of A and B that would make Al(叭vec] 1+8[(v)wec]2-4[^(i)]-3[^(j), You may assume that each grid line is 1 unit away from the next. y-axis 2 x-axis 1pts Subit Aswe Tries 0/15 For the vectors shown below, determine the values of A and the acute angle θ2 (in degrees) that would make (v lvec]1+A (v) vec 2" The angle θ2 is between the vector [(v)\vec]2 and the negative y-axis. 3(^() 5.5[^...

  • 6, The two vectors and in the figure have equal magnitudes of 34.0 m, and the...

    The two vectors\(\vec{a}\) and \(\vec{b}\) in the figure have equal magnitude of 26 m and the angles are θ1=28° and θ2 = 103°. Find the x-component of their vector sum r.Find the y-component of their vector sum r.What is the magnitude of their vector sum r?Find the angle that their vector sum r makes with the positive direction of the x-axis.  

  • please answer without using calculus q1 6) For the figure shown in Fig. 1, -20 nC,...

    please answer without using calculus q1 6) For the figure shown in Fig. 1, -20 nC, q2-50 nC, q3-30 nC Size a is 2.4 cm, and side b is 5.5 cm Use the following steps to find the magnitude and direction (angle) of the net force on q3. Be sure to draw and label the forces, showing their directions. Also draw and label the components of the forces In your calculations, show the calculation of the components clearly. q 43...

  • Determine if the capacitor bank should be switched on or off.

    For the two-bus system shown in diagram below, bus 1 is the reference (slack) bus with \(\left|\mathrm{V}_{1}\right|=1.00\) and \(\delta_{1}=0.0^{\circ} .\) At bus \(2,\) the active load is \(4.3 \mathrm{p} . \mathrm{u} .\) and the reactive load is \(-0.63 \mathrm{pu}\) and the line admittance is \(\mathrm{y}_{12}=1\) - \(j 10\) p.u. as shown in the figure. The voltage at bus 2 is to be maintained between 0.95 and 1.05 p.u. a) Determine if the capacitor bank should be switched on or off....

  • 2. General Motion with Unit Vectors and Components An object undergoes the following consecutive displacements: s...

    2. General Motion with Unit Vectors and Components An object undergoes the following consecutive displacements: s (2i +3j +5k)m,s2 (6i-91+ 2krn, and s,-(10i + 8j-km a) Find the resultant displacement in terms of unit vectors and components. b) State the magnitude of the resultant displacement. 2. Suppose a hiker travels 5 km southwest from their camp. Then, the hiker travels 2 km 75° north of east. a) Find the displacement of the hiker form their camp. b) If the hiker...

ADVERTISEMENT
Free Homework Help App
Download From Google Play
Scan Your Homework
to Get Instant Free Answers
Need Online Homework Help?
Ask a Question
Get Answers For Free
Most questions answered within 3 hours.
ADVERTISEMENT
ADVERTISEMENT
ADVERTISEMENT